Completed rec9 solutions
[course.git] / latex / problems / Young_and_Freedman_12 / problem28.23.tex
1 \begin{problem*}{28.23}
2 Four long, parallel power lines each carry $100\U{A}$ currents.  A
3 cross-sectional diagram of these lines if a square, $20.0\U{cm}$ on
4 each side.  For each of the three cases shown in Fig.~28.41, calculate
5 the magnetic field at the center of the square.
6 \end{problem*}
7
8 \begin{nosolution}
9 \begin{center}
10 \begin{asy}
11 import Mechanics;
12
13 real u = 1cm;
14 real a = 1u;
15 real dx = 3u;
16 pen ipen = red+blue;
17
18 real x = 0;
19 real d = a/2;
20
21 Vector Vs[];
22
23 label("(a)", (x,-d), S);
24 Vs.push(Vector((x-d,-d), phi=-90, ipen));
25 Vs.push(Vector((x+d,-d), phi=-90, ipen));
26 Vs.push(Vector((x+d, d), phi=-90, ipen));
27 Vs.push(Vector((x-d, d), phi=-90, ipen));
28 x += dx;
29
30 label("(b)", (x,-d), S);
31 Vs.push(Vector((x-d,-d), phi= 90, ipen));
32 Vs.push(Vector((x+d,-d), phi=-90, ipen));
33 Vs.push(Vector((x+d, d), phi= 90, ipen));
34 Vs.push(Vector((x-d, d), phi=-90, ipen));
35 x += dx;
36
37 label("(c)", (x,-d), S);
38 Vs.push(Vector((x-d,-d), phi= 90, ipen));
39 Vs.push(Vector((x+d,-d), phi= 90, ipen));
40 Vs.push(Vector((x+d, d), phi=-90, ipen));
41 Vs.push(Vector((x-d, d), phi=-90, ipen));
42
43 for (int i=0; i<Vs.length; i+=1)
44   Vs[i].draw();
45 \end{asy}
46 \end{center}
47 \end{nosolution}
48
49 \begin{solution}
50 Note that the magnitude of magnetic field at the center of the square
51 from any corner wire will be
52 \begin{equation}
53   B_w = \frac{\mu_0 I}{2\pi \frac{a}{\sqrt{2}}} \;,
54 \end{equation}
55 where $a=20.0\U{cm}$ is the side length of the square, and
56 $r=a/\sqrt{2}=a\cos(45\dg)$ is the distance from the corner of the
57 square to the center.
58
59 Using our right-hand rules to determine the direction of the magnetic
60 field from each wire
61 \begin{center}
62 \begin{asy}
63 import Mechanics;
64
65 real u = 1cm;
66 real a = 1u;
67 real dx = 3u;
68 pen iUL = red;
69 pen iUR = blue;
70 pen iLL = green;
71 pen iLR = yellow;
72
73 real x = 0;
74 real d = a/2;
75 real dy = 1mm;
76
77 Vector Vs[];
78
79 label("(a)", (x,-d), S);
80 Vs.push(Vector((x-d,-d), phi=-90, iLL));
81 Vs.push(Vector((x,0), dir=-45, iLL));
82 Vs.push(Vector((x+d,-d), phi=-90, iLR));
83 Vs.push(Vector((x,0), dir=45, iLR));
84 Vs.push(Vector((x+d, d), phi=-90, iUR));
85 Vs.push(Vector((x,0), dir=135, iUR));
86 Vs.push(Vector((x-d, d), phi=-90, iUL));
87 Vs.push(Vector((x,0), dir=-135, iUL));
88 x += dx;
89
90 label("(b)", (x,-d), S);
91 Vs.push(Vector((x-d,-d), phi= 90, iLL));
92 Vs.push(Vector((x,0), dir=135, iLL));
93 Vs.push(Vector((x+d,-d), phi=-90, iLR));
94 Vs.push(Vector((x,0), dir=45, iLR));
95 Vs.push(Vector((x+d, d), phi= 90, iUR));
96 Vs.push(Vector((x,0), dir=-45, iUR));
97 Vs.push(Vector((x-d, d), phi=-90, iUL));
98 Vs.push(Vector((x,0), dir=-135, iUL));
99 x += dx;
100
101 label("(c)", (x,-d), S);
102 Vs.push(Vector((x-d,-d), phi= 90, iLL));
103 Vs.push(Vector((x,+dy), dir=135, iLL));
104 Vs.push(Vector((x+d,-d), phi= 90, iLR));
105 Vs.push(Vector((x,-dy), dir=-135, iLR));
106 Vs.push(Vector((x+d, d), phi=-90, iUR));
107 Vs.push(Vector((x,0), dir=135, iUR));
108 Vs.push(Vector((x-d, d), phi=-90, iUL));
109 Vs.push(Vector((x,0), dir=-135, iUL));
110 Vs.push(Vector((x,0), dir=-180)); // Net B field.
111 Vs[Vs.length-1].mag *= 4*Cos(45);
112
113 for (int i=0; i<Vs.length; i+=1)
114   Vs[i].draw();
115 \end{asy}
116 \end{center}
117
118 For both \Part{a} and \Part{b}, the magnetic fields cancel out and the
119 net magnetic field at the center of the square is \ans{zero}.  We can
120 also see this by symmetry, without using the right-hand rule.
121 The \Part{a} configuration is symmetric to $90\dg$ rotations about the
122 square center, so the magnetic field at the center must also be
123 symmetric to $90\dg$ rotations about the square center.  A vector with
124 non-zero length is only symmetric to $360\dg$ rotations, so there
125 cannot be any magnetic field in the center of \Part{a}.  Similarly,
126 the \Part{b} configuration is symmetric to $180\dg$ rotations, so it
127 cannot have magnetic field at the center.  \Part{c} is only symmetric
128 to $360\dg$ rotations, so it can have a magnetic field at the center.
129
130 The magnitude of the magnetic field for \Part{c} is given by the sum
131 of the horizontal components of the corner magnetic fields, since the
132 vertical components cancel out.
133 \begin{equation}
134   B_c = 4 B_w \cos(45\dg)
135     = 4 \cdot \frac{\mu_0 I}{2\pi \frac{a}{\sqrt{2}}} \cdot \frac{1}{\sqrt{2}}
136     = \ans{\frac{2\mu_0 I}{\pi a}} \;.
137 \end{equation}
138 \end{solution}